4
$\begingroup$

This first request may be easy, but the asymptotics for the next step has me scratching my head.

Through an informal inductive argument I have been able to show $$ (1) \quad \sum_{j=0}^{n-1}2^{2m(n-j)}E_{2j}\binom{2n}{2j} = 2\sum_{j=1}^{2^{m-1}}(-1)^j(2j-1)^{2n} , \quad m \ge 2.$$ The $E_{2j}$ are the Euler numbers. In trying to extend this to integers greater than two and I have acquired numerical evidence for $$ (2) \quad \sum_{j=0}^{n-1}r^{2m(n-j)}E_{2j}\binom{2n}{2j} \sim 2 (-1)^r\sum_{j=1}^{r^{m-1}}(-1)^j(rj-1)^{2n} , \quad m,r=2,3,...$$
I have searched for an appropriate identity but have only been able to postulate an asymptotic relationship. Note that because of the factor of $(-1)^r$ and the $r$ in the upper limit of the summation it is likely that (2) applies only to integer $r>1.$ A different proof than mine might help in my own investigations.

My question is: can (2) be proven, and if that is not possible, can (1) be proven without an induction; e.g., generating functions.

$\endgroup$
3
  • 1
    $\begingroup$ Mathematica says that the right hand side is$$2^{2 n+1} (-1)^r r^{2 n} \left((-1)^{r^{m-1}} \left(\zeta \left(-2 n,\frac{r^m+r-1}{2 r}\right)-\zeta \left(-2 n,\frac{r^m+2 r-1}{2 r}\right)\right)-\zeta \left(-2 n,\frac{r-1}{2 r}\right)+\zeta \left(-2 n,1-\frac{1}{2 r}\right)\right)$$ $\endgroup$ Oct 31, 2018 at 4:28
  • $\begingroup$ Have you tried to give a combinatorial interpretation of right- and left- hand side of (1) ? $\endgroup$ Oct 31, 2018 at 11:13
  • $\begingroup$ @ Maurizio Moreschi I would welcome a combinatorial interpretation, but it's unlikely I could come up with one on my own. $\endgroup$
    – skbmoore
    Oct 31, 2018 at 15:06

1 Answer 1

2
$\begingroup$

Let $$T_{k,n}(a,d) =a^k -(a+d)^k +(a+2d)^k +\cdots + (-1)^{n-1}\bigl(a+(n-1)d\bigr)^k.$$ Equation (7.4) of F. T. Howard, Sums of powers of integers via generating functions, Fibonacci Quarterly 34 (1996), no. 3, 244–256, is $$T_{k,n}(a,d) = \frac{d^k}{2}\left[(-1)^{n-1}E_k\left(\frac ad +n\right) +E_k\left(\frac ad\right) \right],\tag{$*$}$$ where $E_k(z)$ is the Euler polynomial defined by $$\frac{2e^{xz}}{e^x+1}=\sum_{k=0}^\infty E_k(z) \frac{x^k}{k!}.$$ In terms of the Euler numbers, we have $$E_k(z) = \sum_{i=0}^k \binom ki \frac{E_i}{2^i}\left( z-\frac12\right)^{k-i}$$ and the Euler numbers are given by $E_n = 2^n E_n (\tfrac12)$. (Note that $E_n=0$ for $n$ odd.) If we take $a=1, d=2$ in $(*)$ and simplify we get $$2\bigl(1^k -3^k +\cdots +(-1)^{n-1} (2n-1)^k \bigr)= (-1)^{n-1}\sum_{i=0}^{k-1}\binom ki E_i (2n)^{k-i}+\bigl((-1)^{n-1} +1\bigr) E_k,$$ which generalizes (1).

Taking $a=r-1, d=r$ in $(*)$ will give a formula for the right side of $(2)$ but it doesn't seem to be close to the left side of $(2)$.

$\endgroup$

Your Answer

By clicking “Post Your Answer”, you agree to our terms of service and acknowledge you have read our privacy policy.

Not the answer you're looking for? Browse other questions tagged or ask your own question.